nervous system practice questions

Réussis tes devoirs et examens dès maintenant avec Quizwiz!

Ganglia associated with efferent nerve fibers mostly contain cell bodies of ______. A)autonomic sensory neurons B)somatic sensory neurons C)autonomic motor neurons D)somatic motor neurons

C

Which of the following structures is severed as a treatment for epilepsy to prevent the spread of epileptic activity? a)diencephalon b)corpus callosum c)cerebral cortex d)basal ganglia e)amygdala

b

Which of the following suprarenal structures produces glucocorticoids? zona reticularis zona fasciculata suprarenal medulla zona glomerulosa

b

Which of the following syndromes occurs as a result of overproduction of thyroxine? Cushing's disease Graves' disease Addison's disease gigantism

b

Which of the following terms is a synonym for eyelids? Commissures Palpebrae Caruncles Tarsal plates

b

A synonym for the autonomic nervous system reflects its major function. The synonym is ______. the peripheral nervous system the central nervous system the involuntary nervous system the somatic nervous system

c

Which of the CNS neuroglial cells plays a role in maintaining the blood-brain barrier and repairing damaged neural tissue? a)Satellite cells b)Microglia c)Oligodendrocytes d)Astrocytes

d

The location at which a neuron interacts with its target cell (another neuron or a muscle cell or other effector cell) is called the __________. A)synapse B)junction C)connection D)axoaxonic target

A

What effect would blocking voltage-regulated calcium channels at a cholinergic synapse have on synaptic communication? A)Communication would cease. B)Communication would be enhanced. C)Communication would be misdirected. D)Communication would continue as before.

A

Which of the following brain systems is heavily involved in mediating emotional responses? A)Limbic system B)Reticular formation C)Reticular activating system D) Both a and b

A

Why are spinal nerves classified as "mixed" nerves? A)They contain both afferent and efferent fibers B)They contain the cell bodies of sensory neurons C)They carry mixed or conflicting information D) Axons of the ventral root carry sensory information to the spinal cord and axons of the dorsal root control somatic and visceral effectors

A

Why can damage to the medulla oblongata cause death? A)It contains cardiac, vasomotor, and respiratory centers. B)It controls the ANS. C)It contains tracts that process information to and from the cerebellum. D)It contains the headquarters for the reticular activating system.

A

You would expect a neuron that depolarizes to -75 mV would __________. A)return to resting membrane potential without generating an action potential B)fire a much more intense action potential with a peak of +100 mV C)fire a much less intense action potential with a peak of +15 mV D)hyperpolarize

A

______ are stimulated when sound waves vibrate hair cells in the inner ear. A)Mechanoreceptors B)Thermoreceptors C)Photoreceptors D)Nociceptors

A

A patient has damage to several spinal nerves. Which of the major divisions of the nervous system has been damaged? A)Central nervous system B)Peripheral nervous system C)Autonomic nervous system D)All of these are correct

B

A(n) ______ procedure might be performed to test for viral or bacterial meningitis. a)epidural tap B)lumbar tap C)cervical tap D) thoracic tap

B

Ascending tracts transmit ______ impulses while descending tracts conduct ______ impulses. A)motor; sensory B)sensory; motor C)spinal; cranial D)cranial; spinal

B

Compression of which nerve produces the sensation that your leg has "fallen asleep"? A)Sural nerve B)Sciatic nerve C)Femoral nerve D)Tibial nerve

B

Conscious perception of which senses would be affected by damage to the temporal lobes of the cerebrum? A)Olfactory and gustatory B)Olfactory and auditory C)Visual and auditory D)Auditory and gustatory

B

Damage to the lateral geniculate nuclei of the thalamus would interfere with the functions of which of the senses? A)Taste B)Sight C)Touch D)Smell

B

How does the metabolism of a person in deep sleep differ from that of a person in a coma? A)A person in a coma has no brain waves. B)A person in a coma consumes far less oxygen than does a sleeping person. C)A person in a deep sleep can be aroused. D)A person in a coma does not display consciousness signals.

B

In what way(s) is/are temporal and spatial summation the same? A)Both require a rapid succession of stimuli at a single synapse. B) Both are methods by which individual EPSPs combine to result in an action potential. C) Both occur when simultaneous stimuli are applied at different locations, causing a cumulative effect on transmembrane potential. D) All of these are correct.

B

Many water-soluble molecules that are relatively abundant in the blood occur in small amounts or not at all in the extracellular fluid in the brain. Why? A)Blood pressure increases in brain capillaries. B)Blood−brain barrier restricts ion movement. C)Neurons take up most ions. D)Neuroglia remove ions from CSF.

B

Norepinephrine, dopamine, and serotonin are all members of which category of neurotransmitter? A)Hormones B)Biogenic amines C)Amino acids D)Neuropeptides

B

Reflex responses usually remove or oppose the stimulus. Therefore this type of reflexive response would be considered an example of ____. A)A visceral reflex B)Negative feedback C)A reverberating circuit D)Avoidance feedback

B

Severe damage to which of the following brain regions would result in death? A)Cerebral cortex B)Medulla oblongata C)Basal nuclei D)Cerebellum

B

Stronger stimuli are interpreted when the CNS receives ________ action potentials. A)higher magnitude B)more frequent C)lower magnitude D)more infrequent

B

The basis for differentiation between gray matter and white matter in the CNS is the presence of _______ in white matter. A)unmyelinated fibers B)myelinated fibers C)Schwann cells D)ependymal cells

B

The relationship between a neurotransmitter, such as NE and cAMP is ____, which is enhanced by a "link" called a ___ ? A) They are both amino acids/carboxyl group B) 1st messenger and 2nd messenger/G protein C) Neurotransmitters/2nd messenger D) EPSP and IPSP/1st messenger

B

Where is the cerebrospinal fluid that surrounds the spinal cord located? A) Epidural space B) Subarachnoid space C) Above the dura mater D) Between the pia mater and the nervous tissue of the brain

B

Which area helps plan movements? A)Primary motor cortex B)Premotor cortex C)Broca's area D)Frontal eye field

B

Which portion of the spinal cord would be affected by a disease that damages myelin sheaths? A) Sulci B) Columns C) Grey horns D) Fissures

B

"Nerves" are found in the ______ while "tracts" are found in the ________. A)PNS; CNS B)CNS; PNS C)brain; legs and arms D)spinal cord; brain

A

A person with polio has lost the use of his leg muscles. In which area of his spinal cord would you expect the virus-infected motor neurons to be? A)anterior gray horns B) lateral gray horns C) posterior gray commissure D) anterior white commissure

A

An intensely painful stimulus is distinguished from a mildly painful one by _______. A)a higher frequency of action potentials initiated B)higher-magnitude action potentials C)an increased number of receptors stimulated D)all of the above

A

Complete the following analogy: Electrical wire is to electrical insulating tape as peripheral neurons are to ________. A)Schwann cells B)oligodendrocytes C)astrocytes D)ependymal cells

A

Damaged basal nuclei, as in Parkinson's disease, result in ______. A)the inability to monitor the intensity of movements B) memory loss C) inability to store memories D) both a and c

A

Depolarization of the neuron refers to __________. A)a reduction in membrane potential B)an increase in membrane potential C)an increased negativity of the neuron interior D)both b and c

A

During the events involved in information transfer across a chemical synapse, which of the following steps would be directly interrupted by exposing a neuron to a calcium channel blocker? A)Neurotransmitter exocytosis from the presynaptic neuron B)Depolarization of the presynaptic neuron C)Binding of neurotransmitters to the postsynaptic membrane D)Depolarization of the postsynaptic membrane

A

How would decreased diffusion across the arachnoid granulations affect the volume of cerebrospinal fluid in the ventricles? A)Volume would increase. B)Volume would decrease. C)Volume would remain the same. D)Volume would fluctuate erratically.

A

How would the stimulation of the muscle spindles involved in the patellar (knee-jerk) reflex by gamma motor neurons affect the speed of the reflex? A)Quicker reflex response B)Slower reflex response C)Graded reflex response D)Inhibited reflex response

A

If a patient has persistent numbness in the medial two fingers, which nerve of the brachial plexus has been damaged? A)Ulnar nerve B)Median nerve C)Radial nerve D)Musculocutaneous nerve

A

In a living neuron, which component provides resistance to current flow? A)the membrane B)the axon C)the cytoplasm D)the ions

A

In addition to its vital role in maintaining overall body homeostasis, which brain region synthesizes two major body hormones? A)Hypothalamus B)Epithalamus C)Interthalamic adhesion D)Thalamus

A

What brain regions make up the brain stem? A)The hypothalamus, the thalamus, and the pons B)The diencephalon and the mesencephalon C)The mesencephalon, the pons, and the medulla oblongata D)The pons, the cerebellum and the medulla oblongata

C

Which brain disorder is characterized by memory loss, shortened attention span, disorientation, and eventual language loss? A)Huntington's disease B)Parkinson's disease C)Alzheimer's disease D)Cerebrovascular accidents

C

Which component of the diencephalon is responsible for integration of the nervous and endocrine systems? A)Thalamus B)Pituitary gland C)Hypothalamus D)Mamillary bodies

C

Which nuclei in the medulla oblongata are responsible for relaying somatic sensory information to the thalamus? A)Nuclei of superior and inferior colliculi B)Nuclei of cranial nerves VIII-XII C)The nucleus gracilis and the nucleus cuneatus D)The solitary nucleus and olivary nucleus

C

A person is most likely to have a nightmare during which stage of sleep? A)REM B)NREM 1 C)NREM 2 D)NREM 3 and 4

D

After injuring her back, Tina exhibits a positive Babinski reflex. What does this imply about Tina's injury? A)Possible paraplegia B)Possible damage of sensory tracts to foot C)Possible damage of ascending tracts in spinal cord D)Possible damage of descending tracts in the spinal cord

D

After suffering a stroke, a patient is unable to speak, but can understand what is said to him and can understand written messages. Which part of his brain has been affected by the stroke? A)Wernike's area in the parietal lobe B)General interpretive area of the temporal lobe C)Primary visual cortex in the occipital lobe D)Broca's area in the frontal lobe

D

Damage to the amygdaloid body would interfere with regulation of what division of the autonomic nervous system? A)Peripheral B)Somatic C)Sympathetic D)Parasympathetic

D

Damage to the long thoracic nerve would cause paralysis to which muscle(s)? A)Deltoid and teres minor muscles B)Rhomboids and levator scapulae muscles C)Latissimus dorsi muscle D)Serratus anterior muscle

D

Damage to which root of a spinal nerve would interfere with motor function? A)Posterior root B)Anterior root C)Dorsal root D)Ventral root

D

Each neuron in our bodies has a life span of _________. A)120 days B)10 years C)10 months D)an average human life span

D

Excitatory neurotransmitters cause ___ and inhibitory neurotransmitters cause ___? A)Repolarization/return to resting potential B)Suppression of generation of action potentials/promote generation of action potentials C)Synaptic fatigue/synaptic delay D) Depolarization/hyperpolarization

D

Identify the false statement. A)Temporal summation occurs with graded potentials. B)EPSPs and IPSPs can summate spatially. C)EPSPs and IPSPs can summate temporally. D)Spatial summation occurs with action potentials.

D

In what way(s) is the cranial dura mater structurally distinct from the spinal dura mater? A)It is formed of two layers. B)Some portions extend into the cranial cavity as dural folds. C)It contains dural sinuses. D)All of these are differences from spinal dura mater.

D

Maintenance of the resting membrane potential is possible due to __________. A)ATP expenditure of the neuron B)the actions of the sodium-potassium pump C)potassium ions spontaneously crossing to the interior of the neuron D)both a and b

D

Reflex movements of the eyes, head and neck are controlled by which area of the mesencephalon? A)Substantia nigra B)Red nucleus C)Inferior colliculi D)Superior colliculi

D

Someone with paralysis of the anterior thigh muscles would have damage at which spinal segments? A)L4-S4 B)T12-L1 C)L2-L4 D)L4-S2

D

The ______ of the brain is the location of our conscious mind. A)Brodmann area B)cerebellum C)brain stem D)cerebral cortex

D

What effect would decreasing the concentration of extracellular potassium ions have on the transmembrane potential of a neuron? A)Repolarization B)Hypopolarization C)Decreased transmembrane potential D)Hyperpolarization

D

When you first enter a movie theater, the smell of popcorn overwhelms you. Once the movie has ended, however, you don't notice the popcorn smell. You have experienced ________. A)phasic sensation B)tonic sensation C)generator sensation D)adaptation

D

Which receptors detect painful stimuli and where do they send the information? nociceptors -spinothalamic tract- thalamus nociceptors -pyramidal tract -thalamus proprioceptors -spinothalamic tract- thalamus proprioceptors- pyramidal tract - thalamus

a

The _______ of sound waves is interpreted as differing pitches, whereas the _______ of sound waves is interpreted as loudness. frequency; amplitude quality; decibels amplitude; frequency decibels; quality

a

The cornea is actually part of the ______ layer of the eye. fibrous vascular sensory scleral

a

The majority of preganglionic parasympathetic fibers are housed within which cranial nerve? Vagus Glossopharyngeal Facial Trigeminal

a

What are the two types of ACh receptors located on postsynaptic membranes of parasympathetic neurons? a)Nicotinic and muscarinic receptors b)Alpha and Beta receptors c)Beta1 and Beta2 receptors d)G proteins and 2nd messengers

a

What does sympathetic stimulation of the smooth muscle around the pupil do? makes the outer radial muscle layer contract so the pupil dilates (gets bigger) makes the outer radial muscle layer contract so the pupil constricts (gets smaller) makes the outer radial muscle layer relax so the pupil dilates (gets bigger) nothing; it has no function

a

Which hormone is produced in the hypothalamus? ADH ACTH LH GH

a

Which is the best sequence regarding a nerve impulse over a reflex arc? a)receptor, sensory neuron, integrating center, motor neuron, effector b)effector, sensory neuron, integrating center, motor neuron, receptor c)receptor, motor neuron, integrating center, sensory neuron, effector d)receptor, integrating center, sensory neuron, motor neuron, effector

a

Which of the choices below includes three primary endocrine organs? pituitary gland, pancreas, and gonads heart, liver, and stomach small intestine, kidney, and skin parathyroid gland, thymus, and skin

a

Which of the following homeostatic imbalances of the ANS is a life-threatening condition involving uncontrolled activation of autonomic neurons? Autonomic dysreflexia Raynaud's disease Hypertension Orthostatic hypotension

a

Which of the following hormones primarily stimulates osteoclasts and reduces urinary excretion of calcium ions? parathyroid hormone (PTH) calcitonin thymosin thyroxine (T4)

a

Which of the following motor pathways remains uncrossed along its length? a)Vestibulospinal tract b)Tectospinal tract c)Anterior corticospinal tract d)Lateral corticospinal tract

a

Which of the following organization of neuronal pools operates on a feedback mechanism? a)reverberation b)serial processing c)convergence d)divergence

a

Which of the following statements regarding "reverberation" in neural circuits is CORRECT? a)Reverberation cannot occur within a single neuronal pool. b)Reverberation circuits use negative feedback to function, in which collateral axons extend back toward the source of an impulse and further stimulate the presynaptic neurons. c)Once a reverberating circuit has been activated, it will continue to function until synaptic fatigue or inhibitory stimuli break the cycle. d)Reverberation is a feedback mechanism, which can only be excitatory.

c

Which of the following statements regarding gustation is FALSE? The typical adult has more than 10,000 taste buds. The mechanism of gustatory reception appears to parallel that of olfaction. Stimulation of the gustatory cell results in action potentials in the efferent fibers. An individual's tasting abilities change with age.

c

Which of the following structures are receptors that monitor the position and movement of skeletal muscles and joints? a)exteroceptors b)interoceptors c)proprioceptors d)neuroeffectors

c

Which of the following structures produces a hormone responsible for stimulating red blood cell production? Stomach Heart Kidney Skin

c

Which of the following types of receptors are activated when the cell membranes are distorted due to pressure such as touching an object? proprioceptors nociceptors mechanoreceptors chemoreceptors

c

Which of these is a fact pertaining to REM sleep? Activity of the cerebral cortex is at a minimum . Energy use declines by about 30%. There is intense inhibition of somatic motor neurons. BP declines by about 30%.

c

Which pancreatic cell type is paired with the hormone it produces? Alpha cells/pancreatic polypeptide F cells/a hormone identical to growth hormone-inhibiting hormone Beta cells/insulin Delta cells/glucagon

c

Which structure contains a modified sympathetic ganglion? a)Sympathetic chain ganglia b)Splanchnic nerves c)Adrenal medulla d)Collateral ganglia

c

Which structures are the visual reflex centers controlling the extrinsic muscles of the eyes? Lateral geniculate bodies Pretectal nuclei Superior colliculi Suprachiasmatic nuclei

c

Which structures comprise the major subdivisions of the bony labyrinth? Anterior, posterior, and lateral semicircular ducts Middle ear, inner ear, and membranous labyrinth Vestibule, three semicircular canals, and the cochlea Vestibule, saccule, and utricle

c

Which type of lingual papillae do not contain taste buds? What is their function? Circumvallate papillae/perception of water molecules Umami papillae/perception of glutamate Filiform papillae/provide friction Fungiform papillae/appreciation of texture

c

Which type of tactile receptor gives you the most information about location, size, and shape of the source of stimulation? a)Root hair plexus b)Free nerve endings c)Tactile (Merkel's) discs d)Lamellated (Pacinian) corpuscles

c

Why does the smell of formaldehyde seem to dissipate during the time you spend in the dissection lab, yet you are able to detect the cologne of someone who just entered the lab? Olfactory receptors are destroyed by formaldehyde. Olfactory receptors adapt to the smell of formaldehyde. Central adaptation has occurred, though olfactory receptors retain sensitivity to other smells. None of these is correct.

c

__________ is the adrenal hormone responsible for maintaining appropriate blood sodium levels. Cortisol DHEA Aldosterone Epinephrine

c

A functional unit is the smallest structure that can carry out the functions of the system. Which of the following pairings are correct? a)kidney: nephron b)nervous system: neurotransmitter c)keletal muscle: sarcomere d)Both A and C e)All of the above

d

A major difference between neurotransmitters and hormones is that hormones are secreted ________. directly onto their target cell into the cerebrospinal fluid into ducts into the blood

d

A malfunction of which of the following cells can cause a deficiency in the absorption of glucose into the cells of the body? delta cells alpha cells F cells beta cells

d

Blood levels of hormone are kept within very narrow ranges by ______ mechanisms. humoral neural hormonal negative feedback

d

Each of the following regarding the blood-brain barrier is true EXCEPT a)it provides a functional barrier between blood and interstitial fluid in the brain. b)renders blood capillaries highly selective but still permeable. c)foot processes from astrocytes contribute. d)gap junctions exist between endothelial cells of the capillaries. e)numerous membrane carriers and transporters are embedded in endothelial cell apical and basal membranes.

d

For what anatomical reason does the left side of the brain control motor function on the right side of the body? a)Motor function is controlled by a system of two neurons b)Motor function is controlled by cranial reflexes c)The motor area devoted to a particular area becomes larger d)Decussation of axons

d

Glaucoma is a disorder in which _______. the scleral venous sinus becomes blocked intraocular pressure increases above 16 mm Hg the retina is compressed and its blood supply is blocked all of the above occur

d

How does stimulation of muscarinic receptors in cardiac muscle affect the heart? a)Increased cardiac tone b)Increased heart rate c)Decreased cardiac contractile force d)Decreased heart rate

d

How does the hypothalamus regulate nervous and endocrine activities? It secretes regulatory hormones that influence endocrine cells in the pituitary. It has autonomic centers that exert direct neural control over endocrine cells of the adrenal medullae. Hypothalamic neurons synthesize hormones and transport them along axons within the infundibulum. All of these are correct.

d

In order to turn the eye straight down, the actions of the _______ muscle(s) is (are) required. superior rectus and superior oblique inferior rectus inferior rectus and lateral rectus inferior rectus and superior oblique

d

Iodine deficiency in the diet produces what symptom? Increased rate of metabolism Increased body temperature Exaggerated response to physiological stress Goiter

d

Limited regeneration occurs in the central nervous system, partially due to the fact that scar tissue is formed by ______, which also release chemicals that block axonal regrowth. a)ependymal cells b)microglia c)oligodendrocytes d)astrocytes

d

Moving inward from the bones of the skull you encounter a)Dura mater → subdural space → pia mater → subarachnoid space → arachnoid membrane → brain b)Subdural space → dura mater → pia mater → subarachnoid space → arachnoid membrane → brain c)Arachnoid membrane → subarachnoid space → dura mater → subdural space → pia mater → brain d)Dura mater → subdural space → arachnoid membrane → subarachnoid space → pia mater → brain

d

Neurons that have two processes attached to the cell body are called ____. a)multipolar b)microglia c)pseudounipolar d)bipolar

d

Neurons that lie entirely within the CNS are called a) sensory neurons. b)somatic motor neurons. c) efferent neurons. d) interneurons. e) afferent neurons.

d

Occasionally, a female patient experiences hirsutism, or excessive facial hair growth, yet blood tests reveal that her levels of testosterone are normal for a female. Another cause could be hypersecretion of _______. catecholamines mineralocorticoids glucocorticoids gonadocorticoids

d

Someone is gently brushing your arm with a feather. Which type of receptor is stimulated? a)Tactile (Meissner's) corpuscles b)Tactile (Merkel's) discs c)Root hair plexuses d)All of the above are correct

d

Taste is 80% smell. The relationship between smell and taste is evidenced by the fact that both sensations are a type of ________. photoreception baroreception mechanoreception chemoreception

d

The Nernst equation returns a value of ~ −90 mV yet the average resting membrane potential for neurons is actually ~ −70 mV. Why the difference? a)The Nernst equation only includes membrane permeability to K+. b)Proteins trapped within the neuron are contributing to resting membrane potential. c) Na+ also contributes slightly to resting membrane potential. d) Both A and C.

d

The cerebellum monitors proprioceptive information for which two senses? a)Visual and hearing b)Hearing and postural information c)Taste and smell d)Vestibular and visual

d

The crossed-extensor reflex is important when ______. a) someone taps on your patellar tendon b) a bright light shines in your eyes c) you overextend your leg during the long jump d) you step barefoot on a sharp object

d

The effects of two antagonistic hormones oppose each other. Which of the following hormones is an antagonist to parathyroid hormone? thyroid hormone somatostatin aldosterone calcitonin

d

The hormone produced by the heart ultimately causes the body to lose ____. water, calcium ions, and sodium ions water and potassium ions potassium and sodium ions water and sodium ions

d

The hypophyseal portal system is the term for the vessels between the ____ and the ____. thalamus; posterior lobe of pituitary hypothalamus; posterior lobe of pituitary hippocampus; anterior lobe of pituitary hypothalamus; anterior lobe of pituitary

d

The pyramids are a region within the medulla that are responsible for which of the following? a)Control of eye movement b)Transfer of information from the cerebellum to the cerebrum c)Control of blood pressure, breathing, swallowing, and vomiting d)The fact that each side of the brain controls the opposite side of the body

d

The secretion of ________ helps regulate our circadian rhythms. estrogen testosterone thyroid hormones melatonin

d

Which gland releases glucocorticoids such as cortisol in response to increased adrenocorticotropic hormone from the anterior pituitary? thyroid gland adrenal medulla pituitary gland adrenal cortex

d

Which of the following is/are correct concerning structural classifications of neurons? a)Multipolar neurons control skeletal muscles. b)Bipolar neurons function in the senses of hearing, smell and vision. c)Anaxonic neurons have indistinguishable axons and dendrites. d)All of the above are correct.

d

Which of the following molecules act as second messengers? cAMP Ca2+ Inositol triphosphate All of the above

d

Which of the following neural circuits is characterized by a system in which information is relayed in a stepwise sequence, from one neuron to another, or from one neural pool to the next? a)convergence b)divergence c)parallel processing d)serial processing

d

You are attending an incredibly loud rock concert. What is the reaction of your tensor tympani to the noise? It extends to allow free movement of the auditory ossicles, thus preventing hearing loss. It extends to prevent all movement of auditory ossicles. It contracts to reduce movement of the stapes. It contracts to pull the malleus medially and stiffens the tympanic membrane.

d

_________ trigger(s) secretion of aldosterone. Increased K+ Angiotensin II ANP Both a and b

d

antagonism synergism agonism permissiveness

d

Which of the following pairings is NOT correct? a)Midbrain; relays signals and visual reflexes b)Medulla oblongata; contains control centers for blood pressure and breathing c)Hypothalamus; contains integrating centers for homeostasis d)Cerebellum; coordinates execution of movement e)Thalamus; relays and modifies information going to and from the cerebellum

e

A blood sample shows elevated levels of somatomedins. Which pituitary hormone will be elevated as well? Thyroid stimulating hormone Growth hormone Oxytocin Adrenocorticotropic hormone

b

A ganglion in the PNS is equivalent to ________ in the CNS. a)a nerve cord b)a nucleus c)white matter d)nerve fiber

b

How many motor neurons are required to conduct an action potential from the spinal cord to smooth muscles in the wall of the intestine? a)One b)Two c)Four d)Six

b

How would a chemical that blocks sodium channels in neuron cell membranes affect its ability to depolarize? a)It would enhance depolarization. b)It would completely inhibit depolarization. c)It would slow depolarization. d)It would have no effect on depolarization.

b

If levels of a certain hormone are high, which of the following will happen? A second messenger brings more of that hormone into the cell. There will be a decrease in the number of cell receptors for that hormone and cells become less sensitive to it. A G protein will bind to that hormone. There will be an increase in the number of cell receptors for that hormone and cells become more sensitive to it.

b

If the ECF K+ concentration increases from 3 mM to 5 mM, what happens to the resting membrane potential of cells? a) It becomes more negative. b)It becomes less negative. c)It does not change.

b

If transduction does not occur, what do you perceive about a stimulus? a) It is stronger than usual. b) It is as though the stimulus did not take place and you feel nothing. c)UV light was not converted into part of the visible spectrum. d)It is weaker than usual.

b

Impulses go from the gustatory cells to the cerebral cortex via all of the cranial nerves except for ____. N VII N V N X N IX

b

In the central nervous system, a collection of neuron cell bodies with a common function, and with a discrete anatomical boundary, is called a ______. a)ganglion b)nucleus c)nerve d)center

b

In the inner ear, the complex of densely packed calcium carbonate crystals and gelatinous matrix is called a (an): endolymphatic duct. otolith. macula. ossicle.

b

Most neurons cannot undergo cell reproduction because they lack ____. a)nuclei b)centrosomes c)mitochondria d)ribosomes

b

Nerve impulses would travel most quickly through axons of which of the following characteristics? a)0.5 μm diameter, unmyelinated b)20 μm diameter, myelinated c)20 μm diameter, unmyelinated d)0.5 μm diameter, myelinated

b

Neurons generally repolarize once the membrane potential reaches approximately +30 mV because __________. a)voltage-gated sodium channel gates close b)voltage-gated potassium channel gates open c)chemically gated sodium channels open d) voltage-gated calcium channel gates open

b

Nociceptors are _____ receptors. To what categories of sensations are nociceptors sensitive? a)Thermoreceptors/extremes of temperature and changes in vibration b)Pain/extremes of temperature, mechanical damage and chemicals released from injured cells c)Pain/changes in pressure or stretch in muscles d)Chemoreceptors/changes in concentration of chemicals or compounds

b

Sympathetic fibers innervating the iris of the eye cause which response? Constriction Dilation Papillary decrease Rapid blinking

b

The ability of the nervous system to change activity at a synapse is called a)saltatory conduction. b) synaptic plasticity. c)divergence. d)convergence.

b

The dorsal roots along the spinal cord contain _______ neurons, while the dorsal rami contain ________ neurons. a) motor; motor b) sensory; mixed c) sensory; motor d)motor; mixed

b

What effect does a decrease in phosphodiesterase activity in photoreceptor cells have on vision? Increased neuron response to photons Decreased neuron response to photons Increased visual acuity Increased light sensitivity

b

What is a function of the eustachian tube, which connects the middle ear with the pharynx? signal transduction of pitch maintenance of pressure on the middle ear signal transduction of frequency signal transduction of loudness

b

What is the role of the auditory ossicles in hearing? They play a major role in equilibrium. They act as levers that conduct vibrations to the inner ear. They vibrate against the round window and distort the perilymph in the vestibular duct. They play no role in hearing.

b

What is unusual about the posterior spinocerebellar tracts when compared with other ascending pathways? a)The decussation of axons in the medulla innervate motor neurons on the opposite side of the body. b)Axons enter the posterior spinothalamic tract on the same side of the body. c)They are the only tracts to carry information about referred pain. d)They carry information along visceral sensory pathways.

b

When an infant suckles at his mother's breast, the mother's neurohypophysis releases oxytocin. This is an example of ________ stimulation. humoral neural hormonal negative feedback

b

Where are the sensory receptors that respond to rotational movements of the head? In the utricle and saccule In the anterior, posterior, and lateral semicircular ducts In the cochlear duct In the vestibular duct and tympanic duct

b

Which cells participate in the repair of damaged nerves in the peripheral nervous system by forming a solid cellular cord that follows the path of the original axon? a)satellite cells b)Schwann cells c)ependymal cells d)oligodendrocytes

b

Which of the following brain imaging techniques uses radioactive-labeled glucose? a)Electroencephalography (EEG) b)Positive emission tomography (PET) c)Functional magnetic resonance imaging (MRI) d)Horseradish peroxidase (HRP)

b

Which of the following describes the general sequence of events at a neuromuscular synapse? 1 - Receptor binding may lead to the generation of an action potential in the axon (if the postsynaptic cell is a neuron) or sarcolemma (if the postsynaptic cell is a skeletal muscle fiber). 2 - Arrival of the action potential at the terminal bouton triggers release of neurotransmitter from secretory vesicles, through exocytosis at the presynaptic membrane. 3 - The neurotransmitter molecules are either enzymatically broken down or reabsorbed. 4 - Receptor binding results in a change in the permeability of the postsynaptic cell membrane. 5 - The neurotransmitter diffuses across the synaptic cleft and binds to receptors on the postsynaptic membrane. a)2-5-1-3-4 c) 5-2-1-3-4 b)2-5-4-1-3 d) 1-2-5-3-4

b

Which of the following hormones is released in direct response to neural stimulation? insulin epinephrine parathyroid hormone calcitonin

b

Which of the following is NOT one of the common signals that alter hormone secretion from endocrine glands? neural stimulation contractions in muscles surrounding the gland tropic hormones blood levels of metabolites

b

Which of the following is a consequence of age-related changes in the synaptic organization of the brain? There is a reduction in rate of blood flow in arteries around the brain. The number of dendritic branches and interneuronal interconnections decrease. Neurons in the brain accumulate abnormal intracellular deposits. The gyri are narrower and sulci are wider.

b

Which of the following is an effect of glucagon? Accelerated glucose uptake and utilization Breakdown of triglycerides in adipose tissue Stimulation of amino acid absorption and protein synthesis Stimulation of glycogen formation

b

Which of the following is an indirect effect of PTH? Increasing osteoclast activity Increasing calcium absorption in the intestines Increasing calcium reabsorption in the kidney Activating vitamin D

b

Which of the following is correct concerning dual innervation? a)It allows nerves to maintain a background level of activity. b)Organs receive information from both sympathetic and parasympathetic divisions of the ANS. c)Where it exists, the two divisions have the same effects. d)All of the above are correct.

b

Which of the following is not an action of glucagon? Release of glucose to the blood by liver cells Transport of glucose into most body cells Synthesis of glucose from lactic acid Breakdown of glycogen

b

Which of the following is the CORRECT ORDER of steps in the production of an auditory sensation? a)6-2-5-4-1-3 b)6-5-4-2-1-3 c)6-5-4-1-2-3 d)6-4-5-2-1-3

b

Which of the following nervous system organizational pairings are correct? a) Efferent neurons of the CNS: somatic motor and autonomic neurons b) Efferent neurons of the PNS: somatic motor and autonomic neurons c)Afferent neurons of the PNS: somatic motor and autonomic neurons d) Afferent neurons of the CNS: somatic motor and autonomic neurons

b

Which of the following receptors provide information about the position of the body? interoceptors proprioceptors exteroceptors phasic receptors

b

Which of the following statements is TRUE regarding receptive fields? The larger the receptive field, the better the tactile acuity. The smaller the receptive field, the better the tactile acuity. The size of the receptive field determines the amount of pain felt. Receptive fields have no effect on pain sensation or tactile acuity.

b

which structure of a neuron is capable of propagating an electrical impulse? a)Dendrites b)Axon c)Cell body d)Perikaryon

b

During embryonic development, the neural tissue rolls up into a tube, forming a hollow neural tube that forms the a)brain. b)peripheral nervous system. c)central nervous system. d)autonomic nervous system. e)spinal cord.

c

Gynecomastia is an overproduction syndrome associated with which hormone? mineralocorticoids estrogens (female) androgens (male) glucocorticoids

c

How does a β blocker mediate high blood pressure? a)The drug aids parasympathetic stimulation. b)The drug decreases blood volume. c)The drug prevents sympathetic stimulation. d)None of the above is correct.

c

Inhibin primarily targets the ____ and inhibits secretion of ____. posterior lobe of pituitary gland; follicle-stimulating hormone (FSH) anterior lobe of pituitary gland; luteinizing hormone (LH) anterior lobe of pituitary gland; follicle-stimulating hormone (FSH) posterior lobe of pituitary gland; luteinizing hormone (LH)

c

The long bone growth-promoting effects of growth hormone are mediated by _______. somatostatin somatotropic cells insulin-like growth factors insulin

c

What is the effect of renin secretion by the kidneys? Stimulation of Ca2+ and PO4 absorption along the digestive tract Stimulates the production of red blood cells by bone marrow Converts angiotensinogen to angiotensin I Inhibits secretion of ADH and aldosterone

c

What is the function of the auditory tube? It helps amplify sound waves. It allows bacteria to travel from the oropharynx to the middle ear. It permits equalization of pressure on either side of the tympanic membrane. It secretes cerumen.

c

What physiological changes occur in someone who is confronted by a large, vicious, growling dog? a)Decrease in metabolic rate and heart rate b)Decrease in mental alertness and dilation of respiratory passageways c)Increase in heart rate and blood pressure d)Increased action of salivary and digestive glands

c

What results in hair cells sending a signal? mechanically gated Na+ channels letting Na+ rush in mechanically gated Na+ channels letting Na+ rush out mechanically gated K+ channels letting K+ rush in mechanically gated K+ channels letting K+ rush out

c

What term is used to indicate the entire group of related steroids, including calcitriol, cholecalciferol, and various intermediaries? eicosanoid vitamin A vitamin D vitamin C

c

Where do low-frequency sounds vibrate the basilar membrane? Why? Near the round window/perilymph is incompressible Near the oval window/shorter wavelength Far from the oval window/longer wavelength Against the tectorial membrane/pressure waves are created in endolymph of the vestibular duct

c

Which group of messengers is defined as acting on cells within the same tissue? Autocrines Hormones Paracrines Eicosanoids

c

Which of the following is TRUE about one of the two gates in Na+ channels in axons? a)The closing of the activation gate stops the depolarization during an action potential. b)The opening of the activation gate stops the depolarization during an action potential. c)The closing of the inactivation gate stops the depolarization during an action potential. d)The opening of the inactivation gate stops the depolarization during an action potential.

c

Which of the following is TRUE regarding nerves? a)consist of many neurons bundled together and enclosed in a connective tissue sheath b) carry only afferent signals c)consist of axons from a number of different neurons bundled together and enclosed in a connective tissue sheath d)carry only efferent signals

c

Which of the following is a blend of several different, complementary hormones important to the development and maintenance of normal immunological defenses? calcitonin parathyroid hormone thymosin triiodothyronine (T3)

c

Which of the following is a characteristic of neural regeneration? a)After injury, the nucleus remains in its centralized location in order to monitor and control cellular repair. b)Neurons do not return to normal appearance after injury, even if they regain normal function. c)After injury, the chromatophilic substance disappears within the cell body. d)Wallerian degeneration is a key process of neural regeneration involving oligodendrocytes.

c

Which of the following is correct concerning the function of the uvea (vascular tunic)? It stabilizes the shape of the eye and produces collagen fibers. It provides the route for the optic nerve and blood vessels to the eye itself. It regulates the amount of light entering the eye. It secretes and reabsorbs vitreous humor.

c

Which of the following is incorrect concerning chemoreceptors? a)They exhibit peripheral adaptation over a few seconds. b)We are not consciously aware of the sensations they provide. c)Information is routed through the cerebral cortex for control of respiratory and cardiovascular function. d)Chemoreceptive neurons are located in the carotid bodies and aortic bodies

c

Which of the following is not a characteristic of higher order functions? Requires cerebral cortex Involves conscious and unconscious processing Part of the programmed wiring of the brain Subject to modification over time

c

Which of the following is not an effect of thyroid hormones on major organ systems? Increased heart rate and force of contraction Increased sensitivity to sympathetic stimulation Decreased turnover of minerals in bone Stimulation of red blood cell formation

c

Which of the following is the only hormone released by the pars intermedia? follicle-stimulating hormone (FSH) adrenocorticotropic hormone (ACTH) melanocyte-stimulating hormone (MSH) luteinizing hormone (LH)

c

Which of the following sets of conditions accurately describes the anatomy of the parasympathetic nervous system? Thoracolumbar origin; long preganglionic fibers; ganglia in visceral effector organs Craniosacral origin; short preganglionic fibers; ganglia in visceral effector organs Craniosacral origin; long preganglionic fibers; ganglia in visceral effector organs Thoracolumbar origin; short preganglionic fibers; ganglia close to the spinal cord

c

The portion of an axon that communicates with its target cell is the _______. A)dendrite B)axon C)axon terminal D)Cell body

C

A collection of neuron cell bodies outside the CNS is called a _________ and within the CNS is called a _________. a)ganglion; nucleus b)Schwann; oligodendrocyte c)nucleus; ganglion d)Neuronal cell bodies do not exist outside the CNS.

a

A major determinant of a hormone's mechanism of action is ________. whether the hormonal molecule is hydrophobic or hydrophilic its size whether it is rapid acting or slow acting if it activates gene activity or not

a

A physician can determine the approximate location of a spinal injury simply by testing areas of the skin for numbness. These skin areas are referred to as ______. a) dermatomes b) cutaneous areas c) dermozones d) cutaneous zones

a

A neuron under the influence of a neurotransmitter that opens K+ channels will a)become hyperpolarized. b)be more likely to fire an action potential. c)be less likely to fire an action potential. d)have no effect on action potential firing. e)Both A and C.

e

A typical resting membrane potential of a neuron is a)−700 mV. b)+700 mV. c)−7 mV. d)+70 mV. e)−70 mV.

e

As a general rule, the greater the potential difference in charge between two points, the __________ the voltage. A)higher B)lower C)less difference in D)more even

A

Cerebral white matter in each hemisphere is greatest in amount for ______. A)communication between the two hemispheres B)coordination of synchronized movements C)visual association D)olfaction

A

Of the following memory types, which do not require conscious attention? A)Nondeclarative memory B)Declarative memory C)Short-term memory D)Working memory

A

One axon propagates action potentials at 50 meters per second; another carries them at 1 meter per second. Which axon is myelinated? A)Axon that propagates at 50 meters per second B)Axon that propagates at 1 meter per second

A

One pathway in the central nervous system consists of three neurons, another of five neurons. If the neurons in the two pathways are identical, which pathway will transmit impulses more rapidly? A)Pathway with three neurons B)Pathway with five neurons C)They would transmit at the same rate

A

Paul is having a difficult time remembering facts and recalling long-term memories. Which part of his cerebrum is involved? A)Temporal lobe B)Parietal lobe C)Frontal lobe D)Occipital lobe

A

Sodium ion concentration in the cytoplasm of a neuron ________ when its voltage-gated sodium channels open. A)increases B)decreases C)remains unchanged D)decreases tenfold

A

The ______ portion of the spinal cord controls muscle movement. A)ventral B)dorsal C)medial D)posterior

A

What name is given to axons carrying information between the brain and spinal cord, and through which brain regions do they pass? A)Projection fibers; diencephalon, brain stem, and cerebellum B)Commissural; both cerebral hemispheres C)Association fibers; diencephalon, brain stem, and cerebellum D)Arcuate fibers; within a single cerebral hemisphere

A

Which of the following cranial nerves has both sensory and motor functions? A)Facial B)Trochlear C)Hypoglossal D)Accessory

A

The chief contributor to the negative charge of a resting neuron's intracellular fluid is __________. A)a high extracellular concentration of sodium ions B)a high intracellular concentration of anionic proteins C)a high intracellular concentration of potassium ions D)a high extracellular concentration of chloride ions

B

The door slams shut loudly and you flinch. After a few seconds, you realize that your heart is beating very rapidly and forcefully. This response is the result of your ______ nervous system. A)afferent B)autonomic C)central D)somatic

B

What is the primary distinction between chemical and electrical synapses; which type is more rare? A) Electrical synapses involve a neurotransmitter/chemical synapses B) Electrical synapses involve direct connection between cells/electrical synapses C) Chemical synapses involve direct connection between cells/chemical synapses D) Electrical synapses always use ACh/both are equally abundant

B

Which of the cerebellar peduncles allows communication between the cerebellum and pons? A)Transverse cerebellar peduncles B)Middle cerebellar peduncles C)Inferior cerebellar peduncles D)Superior cerebellar peduncles

B

Which of the following embryonic tissues develops into the adult brain? A)Endoderm B)Ectoderm C)Mesoderm D)Neuroderm

B

Which of the following structures would produce less cerebrospinal fluid if it were damaged? A)Subarachnoid space B)Choroid plexus C)Pia mater D)Arachnoid villus

B

Which of the following synapse types is the most rapid? a)Chemical synapse B)Electrical synapse C)Presynaptic synapse D)None of these is more rapid than the others.

B

Which of these is NOT usually a characteristic function associated with the left cerebral hemisphere? A)Performing mathematical calculations B)Analyzing emotional context of a conversation C)Containing the general interpretive and speech centers D)Processing associated with reading, writing, and speaking

B

Which primary brain vesicle is destined to form the cerebellum, pons, and medulla oblongata? A)Prosencephalon B)Rhombencephalon C)Myelencephalon D)Metencephalon

B

Which type of fiber is lightly myelinated and transmits impulses at an average rate of 15 m/s? A)Group A fibers B)Group B fibers C)Group C fibers D)Group D fibers

B

Which type of gated channel responds to physical distortion of the membrane surface? Where is this ability important? A)Chemically regulated channels/in dendrites B)Mechanically regulated channels/sensory receptors C)Voltage-gated channels/axons of multipolar and unipolar neurons D)Passive channels/where the channels must remain open

B

Which type of potential does not decay with distance? A)Graded potential B)Action potential C)Generator potential D)Both a and b

B

A patient suffers a head injury that damages her primary motor cortex. Where is the primary motor cortex and what functions will be lost due to the injury? A)Parieto-occipital sulcus/control over involuntary movements B)Postcentral gyrus/voluntary movements controlled by the specific regions damaged C)Precentral gyrus/voluntary movements controlled by the specific regions damaged D)Insula/ability to categorize items

C

An anesthetic blocks the function of the dorsal rami of the cervical spinal nerves. Which areas of the body will be affected? A)Skin and muscles of the back of the neck B) Skin and muscles of the shoulders C)Both 1 and 2 D)None of the above

C

At which level of the somatosensory system are conscious decisions made about low-grade touch stimuli? A)Receptor level B)Circuit level C)Perceptual level D)Effector level

C

During a withdrawal reflex, what happens to the limb on the side opposite the stimulus? What is this response called? A)Flexed/stretch reflex B)Adducted/tendon reflex C)Extended/crossed extensor reflex D)Abducted/tendon reflex

C

Generally speaking, opening chloride channels in the postsynaptic membrane will result in an _______. A)excitatory postsynaptic potential B)excitatory presynaptic potential C)inhibitory postsynaptic potential D)inhibitory presynaptic potential

C

If inhibited, which brain system would allow previously filtered stimuli to enter our consciousness? A)Limbic system B)Autonomic system C)Reticular activating system D)Both a and b

C

One of the first somatic reflexes to develop is the suckling reflex. Which type of reflex is this? A)Inherent reflex B)Acquired reflex C)Innate reflex D)Visceral reflex

C

The effects of a neurotransmitter on the postsynaptic membrane depends upon ___. A)The time involved in calcium influx B)ACh being broken down into AChE C)The properties of the receptor, not the nature of the neurotransmitter D) None of these is correct

C

The nervous system has three overlapping functions. Which of the following represents a logical sequence of these three functions? A)Sensory input, motor output, integration B)Motor output, integration, sensory input C)Sensory input, integration, motor output D)Integration, sensory input, motor output

C

The perineurium ______. A)encloses all the fascicles to form a nerve B)surrounds axons and Schwann cells C)binds groups of fibers into bundles D)contains sensory neurons only

C

The phrases "left brain" and "right brain" indicate that the two cerebral hemispheres display a phenomenon called ______. A)left-brain dominance B)right-brain dominance C)lateralization D)cerebral localization

C

The point at which the all-or-none principle of action potential generation is reached is termed the ________. A)peak B)depolarization point C)Threshold D)point of no return

C

What is reinforcement as it pertains to spinal reflexes? A)It allows brief periods of rest between sets of stimuli B)A chronic depression in spinal reflexes due to IPSPs C)An enhancement of spinal reflexes due to postsynaptic neurons entering a state of facilitation D)Strengthening of both the crossed extensor reflex and reciprocal inhibition

C

What is the term used when one presynaptic neuron synapses with multiple postsynaptic neurons ___? When is this useful ____? A)Parallel processing/when information must be relayed in a stepwise fashion B)Convergence/when a signal must be concentrated C)Divergence/when broad distribution of a specific input is important D)All of these are correct

C

When considering the relationship between a structural classification and a functional classification of neurons, it can be said that ____________________. A)all multipolar neurons are interneurons B)all motor neurons are unipolar neurons C)essentially all bipolar neurons are sensory neurons D)unipolar neurons only function as motor neurons

C

Which area of the diencephalon would be stimulated by changes in body temperature? A)Tuberal area of the hypothalamus B)Anterior nuclei of the thalamus C)Preoptic area of the hypothalamus D)Intermediate mass of the thalamus

C

Which cranial nerve nuclei are located in the pons? A)VII, IX, and X B)III, IV, and VI C)V, VI, VII, and VIII D)IX, X, XI, and XII

C

Which imaging technique can localize brain lesions that generate seizures? A)CT scans B)MRI scans C)PET scans D)Cerebral angiography

C

Which of the choices below correctly lists the steps of an action potential? A)Resting potential absolute refractory period relative refractory period B)All-or-none principle stimulus triggers action potential stimulus does not trigger action potential C)Depolarization to threshold activation of sodium channels and depolarization inactivation of sodium channels and activation of potassium channels return to normal permeability D)None of these

C

Which of the following best explains the convolutions and folds present in an adult brain? a) As the brain developed, the differentiation of the primary vesicles into the secondary vesicles occurred as outpocketings and spiral patterns. b) Each of the convolutions present arose from the five secondary vesicles. c) The highly folded convolutions and flexures increase the brain's surface area and allow the brain to fit into the cranium.

C

Which of the following is true concerning the spinal cord? A) The filum terminale is the end of the spinal cord. B)The conus medullaris is a strand of fibrous tissue that helps support the spinal cord. C) The spinal cord of an adult ends between L1 and L2. D) The amount of grey matter in the spinal cord is the least at the cervical and lumbar enlargements.

C

Which of these is NOT a site at which acetylcholine is released at a synapse? A)Neuromuscular junctions with skeletal muscles B)Neuron-to-neuron synapses in the PNS C)Postganglionic synapses in the sympathetic division of the ANS D)All of these are true

C

What are potential consequences of blockage of an interventricular foramen? A)Hugely expanded skulls in infants B)Brainstem stroke C)Damage to and distortion of brain in adults D)1 and 3 are correct

D

What is the clinical significance of dermatomes? A)Damage to descending tracts will be reflected in the dermatomes. B)Location of affected dermatomes provides clues to location of injury along the spinal cord C)Damage to a corresponding spinal nerve will produce loss of sensation the region of skin supplied by a dermatome. D) 2 and 3 are correct.

D

A patient who is hooked up to an EEG shows primarily beta waves. What are they doing? A)They are in deep sleep or have a brain disorder. B)They are in the early stages of sleep. C)They are concentrating on a specific task. D)They are extremely frustrated.

D

A person you know has experienced sleep problems and prolonged depression. Which neurotransmitter may be in short supply? A)Dopamine B)Any of the opioids C)Acetylcholine D)Serotonin

D

A weight lifter is straining to lift a 200-kg barbell. Shortly after he lifts it to chest height, his muscles appear to relax and he drops the barbell. Which reflex has occurred? A)Withdrawal reflex B)Stretch reflex C)Crossed extensor reflex D)Tendon reflex

D

CNS and PNS neurons differ in their regeneration capability, mainly because ______. A)PNS neurons are associated with supporting cells that promote rapid cleanup and regrowth of new axons B)CNS neurons are associated with oligodendrocytes that contain growth-inhibiting proteins C)The statement is not true; neither PNS nor CNS neurons are capable of axon regeneration. D)Both a and b are true.

D

During the relative refractory period __________. A)no action potentials can be generated B)the threshold is substantially elevated exceptionally strong stimuli could C)trigger action potentials D)both b and c apply

D

Professional ballet dancers have a particularly efficient ______. A)cerebral cortex B)medulla oblongata C)basal nucleus D)cerebellum

D

The brain contains _____ ventricles. A)one B)two C)three D)four

D

The pons belongs to which major brain region? A)Cerebellum B)Cerebrum C)Diencephalon D) Brain stem

D

What is the primary function of the cerebellum? A)Controls reflexive movements of the eyes in response to visual stimuli B)Provides awareness of emotional states C)Coordination between voluntary and autonomic functions D)Coordinates rapid, automatic adjustments that maintain balance and equilibrium

D

What symptoms would you expect to observe in an individual who has damage to the basal nuclei? A)Mental slowness and inability to perceive meaning from written symbols B)Conscious perception of touch and erratic muscle contractions C)Epileptic seizures D)Difficulty starting voluntary movements and decreased muscle tone

D

Which cranial nerve is the exception and travels to the thoracic and abdominal cavities? A)Trochlear B)Trigeminal C)Vestibulocochlear D)Vagus

D

Which cranial reflex is stimulated by loud noises and what is the response? A)Tympanic reflex/reduced movement of auditory ossicles B)Vestibulo-ocular reflex/opposite movement of the eyes to stabilize field of vision C)Auditory reflex/eye or head movement D)1 and 3 are correct

D

Which of the following brain regions is a major convergence area for most sensory input before it is sent on to the cerebral cortex? A)Hypothalamus B)Epithalamus C)Interthalamic adhesion D) Thalamus

D

Which of the following brain wave forms, if present in an alert adult, would indicate brain damage? A) Alpha B) Beta C)Theta D)Delta

D

Which of the following is NOT a function of the limbic system? A)Establishing emotional states B)Linking the conscious, intellectual function of the cerebral cortex with unconscious, autonomic functions of the brain stem C)Facilitating memory storage and retrieval D)Directing somatic motor patterns associated with rage, pleasure, and pain

D

Which of the following is true concerning neuromodulators? A)Have short-term effects B)Act immediately without intermediate steps C)Affect neither postsynaptic nor presynaptic membrane D)None of these is correct

D

Which of the following pairs of receptors appear to play complementary roles in hairy and hairless skin? A)Tactile discs and lamellar corpuscles B)Bulbous corpuscles and hair follicle receptors C)Tendon organs and tactile corpuscles D)Tactile corpuscles and hair follicle receptors

D

Which of these is NOT an important factor concerning reflexes? A)They are rapid, automatic responses to stimuli. B)They preserve homeostasis by making rapid adjustments in the functions of organs. C)They oppose potentially harmful changes in the internal or external environment. D)They involve interaction with the cerebrum.

D

While studying for an exam, you reach for a beverage. To extend your arm, your _______ nervous system is activated. A)afferent B)autonomic C)sympathetic D)somatic

D

Why is it impossible for continuous propagation to occur along myelinated axons? A)Myelin increases resistance to the flow of ions across the membrane. B)Only nodes along a myelinated axon can respond to a depolarizing stimulus. C)Continuous propagation uses less energy and fewer sodium ions must be pumped out. D)1 and 2 are correct

D

Why is the blood brain barrier ineffective against alcohol? A)Alcohol is water soluble. B)The blood brain barrier prevents only blood cells from reaching the brain. C)It isn't: Alcohol cannot reach the brain. D)Alcohol is lipid soluble.

D

You've just touched a hot stove. What type of reflex did you experience___ and in what way was your brain involved in the reflex ____? A) Cranial/the somatosensory cortex B) Visceral/only tracts that respond to pain C)Complex/all the sensory tracts D) Spinal/it was not involved at all

D

________ are the most common structural type of neuron in humans. A)Pseudounipolar B)Unipolar C)Bipolar D)Multipolar

D

How does LSD affect the brain stem, hypothalamus, and limbic system? What is the result? Activates serotonin receptors/hallucinations Decreases dopamine secretion/Parkinson-like disorder Decreases availability of serotonin/depression Increases dopamine secretion/schizophrenia

a

How does the presence of a molecule that blocks adenylate cyclase affect the activity of a hormone that produces its cellular effects by way of the second messenger cAMP? It prevents synthesis of the hormone. It enhances the action of the hormone. It increases sensitivity to the hormone. It decreases speed of hormonal changes.

a

If a neuron has receptors for a particular neurotransmitter it will a) respond in a similar way to an agonist of that neurotransmitter. b) respond in a similar way to an antagonist of that neurotransmitter. c)release the same neurotransmitter at its axon terminals. d) only be able to bind that neurotransmitter.

a

All hormones produced by the pituitary gland are ____. peptide hormones steroid hormones amino acid derivative hormones eicosanoid hormones

a

Along with the sympathetic nervous system, the _________ is the other primary mediator of acute stress. adrenal medulla adrenal cortex zona glomerulosa zona reticularis

a

An infection of the ____ gland may result in the formation of a cyst. tarsal lacrimal caruncle palpebral

a

Asthma patients suffer from attacks in which bronchiole smooth muscle constricts, making it hard to breathe. Albuterol, a common asthma medication, works via which of the following steps? Binds to β2-adrenergic receptors to enhance vasodilation Binds to nicotinic receptors and inhibits epinephrine release Binds to and stimulates muscarinic receptors Binds to β2-adrenergic receptors to inhibit vasodilation

a

At which interface in the human eye is refraction the greatest? Between air and the cornea Between aqueous humor and the lens Between the lens and the vitreous body Between the vitreous body and the retina

a

Cells in the ___are insulin independent, which means that they can _____. Brain, kidneys, and digestive tract/absorb glucose without insulin stimulation Liver/produce glucose Pancreas and suprarenal cortex/produce insulin Suprarenal cortex and suprarenal medulla/function without insulin

a

Choose the correct order of events involved in the process of Wallerian degeneration. 1 - The axon sends buds into the network of Schwann cells and starts growing along a cord of Schwann cells. 2 - Fragmentation of the axon and myelin occurs in distal stump. 3 - The axon grows into distal stump and is enfolded by Schwann cells. 4 - Schwann cells form a cord, grow into the cut, and unite the stumps. Macrophages engulf the degenerating axon and myelin. a)2-4-1-3 b)2-1-4-3 c)3-1-4-2 d)4-1-3-2

a

Communication between neurons is not always one-to-one. When a group of presynaptic neurons provide input to a small number of postsynaptic neurons it is called a)convergence. b)divergence. c)plasticity. d)neuromodulation.

a

Deficiencies of which hormones impede normal skeletal development? GH, thyroid hormone, PTH, and reproductive hormones Prolactin, FSH, LH, GH Thyroid hormone, melatonin, PTH, calcitonin GH, TSH, ACTH, FSH

a

Diabetes insipidus develops when the ____ no longer releases adequate amounts of ____. neurohypophysis; antidiuretic hormone neurohypophysis; oxytocin kidneys; renin adenohypophysis; antidiuretic hormone

a

During repolarization of a neuron: a)K+ moves from cell to ECF. b) K+ enters the neuron from ECF. c)membrane potential is becoming less negative. d)there is no movement of K+ as the fiber is in its refractory state. e)Both B and C.

a

During times of stress, elevated levels of _______ often occur, which explains why we get a cold during final exam time. cortisol aldosterone ACTH androgens

a

Dynamic and static equilibrium transduction both use the principle of ________. mechanoreception chemoreception thermoreception photoreception

a

Except in periods of starvation, the only fuel source for the brain is/are _______ so 15% of blood pumped by the heart goes to the brain to supply enough _______ from which to generate ATP. a)glucose; oxygen b)glucose; nutrients c)fatty acids; oxygen d)triglycerides; oxygen e)fatty acids; nutrients

a

For conscious muscle movements, activity of the ______ is required. a) precommand areas b) central program generators c) reflex arc d) indirect system

a

For vision, the photoreceptors are specific for light. Which law states that a given sensory receptor is specific for a particular modality, a particular energy form of a stimulus? law of specific nerve energies law of sensory modalities Weber's law Ohm's law

a

Harry has a brain tumor that is interfering with the function of his hypothalamus. Could this tumor interfere with autonomic function? Why or why not? Yes; hypothalamus regulates ANS. Yes; all brain tumors affect ANS functioning. No; ANS has no relationship with the hypothalamus. No; ANS function is regulated by thalamus.

a

If damage occurs to the Wernicke's area of the brain, a person is unable to understand any spoken or visual information. This condition is known as a)receptive aphasia. b)expressive aphasia. c)Alzheimer's disease. d)schizophrenia. e)anterograde amnesia.

a

In an organism, action potentials are propagated unidirectionally toward the axon ending because __________. a) membranes become nonresponsive due to inactive sodium channels after action potential generation b)action potentials are always propagated in an anterograde fashion regardless of conditions c) action potentials cannot start anywhere except at the axon hillock d) of both a and b

a

In detection of smell, odorants themselves _______. bind to receptors and stimulate opening of ion channels are neurotransmitters are detected by chemoreceptors in the lungs are or do all of the above

a

In order for a hormone to activate a target cell, the target cell must possess _______. a receptor a second messenger the hormone a chaperone

a

Information concerning the general texture of food, together with taste-related sensations of "peppery" or "burning hot" is provided by sensory afferents in which cranial nerve? N V N VII N IX N X

a

Neurotransmitters are important in functioning of ______ synapses. a)chemical b)electrical c)gap d)converging

a

Pinkeye is an infection of the _______ of the eye. conjunctiva lacrimal apparatus cornea palpebrae

a

Predict the outcome of damage to one cord of the left brachial plexus. a) Only a portion of the left limb will suffer reduced function. b) The entire left limb will be paralyzed. c) The entire right limb will be paralyzed. d) Only a portion of the right limb will suffer reduced function.

a

Progestins (especially progesterone) are reproductive hormones released by which structures or cells? corpus luteum of ovaries nurse cells follicular cells of ovaries interstitial cells

a

Receptor A has a circular receptive field with a diameter of 2.5 cm. Receptor B has a circular receptive field of 7.0 cm in diameter. Which receptor allows you to more precisely localize a stimulus? a)Receptor A b)They would provide the same precision of sensory information c)Receptor B d)It would depend upon the location of the receptors

a

Receptor specificity: results from the presence of accessory cells or structures that shield the receptor cell from other stimuli. does not result from the structure of the receptor cell itself. results from sensory coding, which provides information about the strength, duration, variation, and movement of the stimulus. implies that touch receptors may be very sensitive to both pressure and chemical stimuli.

a

Receptors for steroid hormones are commonly located _________. inside the target cell on the plasma membrane of the target cell in the blood plasma in the extracellular fluid

a

Regarding neurological disorders: a)paresthesias may be temporary or permanent. b)loss of feeling, numbness, or tingling sensations may occur after damage to efferent pathways in the central nervous system. c)tension headaches do not accompany severe depression or anxiety. d)muscle weakness does not actually have an underlying neurological basis.

a

Regulation of heart rate at low normal levels is achieved by the ______. parasympathetic division sympathetic division somatic division both A and B

a

Secretions from the corticotropic cells activate cells of the ________, while secretions from the gonadotropic cells affect cells of the ________. adrenal cortex; gonads thyroid; mammary glands gonads; adrenal cortex mammary glands; gonads

a

Some neural circuits utilize positive feedback to produce which of the following? a)reverberation b)serial processing c)convergence d)divergence

a

Sympathetic innervation is predominant during _______.

a

The ____ carries sensation of highly localized (fine) touch, while the ____ provides conscious sensations of poorly localized (crude) touch? a)Posterior column pathway/anterior spinothalamic tract b)Fasciculus gracilis/fasciculus cuneatus c)Spinocerebellar pathway/lateral spinothalamic tracts d)Spinothalamic pathway/spinocerebellar pathway

a

The light-dependent step in vision involves _______. stimulating the transformation in retinal from the 11-cis isomer to the 11-trans isomer maintaining the dark current in the outer segments of the visual receptors opening the cation channels in the outer segments of the visual receptors release of neurotransmitter to the bipolar cells

a

The medial pathway controls ______, whereas the lateral pathway controls _____. a)Gross movements of the trunk and proximal limb muscles/distal limb muscles and precise movement b)Background patterns of movement/transmits motor impulses from the cerebrum to the spinal cord c)Subconscious control of balance and muscle tone/subconscious control of reflex activity d)None of these is correct

a

The receptors for hearing are located in _____, which is filled with ______. The organ of Corti in the cochlear duct/endolymph Hair cells in the vestibular duct/perilymph The tectorial membrane in the tympanic duct/perilymph None of these is correct

a

The removal of the parathyroid glands result in a decrease in the blood concentration of which important mineral? Calcium ions Phosphate ions Sodium ions Potassium ions

a

Visual information from the left half of the combined field of vision _________. Arrives at the visual cortex of the right temporal lobe Arrives at the superior colliculus on the right side Arrives at the visual cortex of the left temporal lobe Arrives at the superior colliculus on the left side

a

What are the qualities of free nerve endings that respond to light touch? small receptive field and slow adaption small receptive field and rapid adaption large receptive field and slow adaption large receptive field and rapid adaption

a

What is critically important about the vascular arrangement of the hypophyseal portal system? It provides efficient means of ensuring that all hypothalamic hormones will reach target cells in the adenohypophysis before being diluted in general circulation . It ensures that all hormones destined for the pituitary are diluted in general circulation before entering the adenohypophysis. It ensures that RH from the hypothalamus synthesizes the correct amount of a specific hormone. None of these is correct.

a

What is the correct path for a signal traveling upward from the spinal cord? a)medulla → pons → hypothalamus → thalamus b)hypothalamus → thalamus → pons → medulla c)pons → medulla → thalamus → hypothalamus d)thalamus → pons → hypothalamus → medulla e)medulla → hypothalamus → thalamus → pons

a

What type of fibers carry information about itch and tickle sensations? a)Unmyelinated Type C fibers b)Myelinated Type A fibers c)All nociceptors d)None of these is correct

a

When the pancreas releases insulin in direct response to blood glucose, this is an example of ________ stimulation. humoral neural hormonal negative feedback

a

Where are the vesicles involved in releasing a neurotransmitter into the synapse located? a)In the area of the presynaptic membrane b)In the area of the postsynaptic membrane c)In the dendrite area d)In the soma area

a

Where do the nerves that synapse in collateral ganglia originate? a)Lateral gray horns of the spinal cord between spinal segments T5 and L2 b)Anterior gray horns of the spinal cord between spinal segments T1 and L2 c)Dorsal gray horns of the spinal cord c)In the brainstem and sacral region of the spinal cord

a

Which cranial nerve supplies taste buds located in circumvallate papillae? Glossopharyngeal nerve Vagus nerve Trigeminal nerve Facial nerve

a

Which of the following statements regarding olfactory receptors is FALSE? When an odorous substance binds to its receptor on the surface of an olfactory cilium, the receptor membrane hyperpolarizes. Olfactory receptor cells are highly modified neurons. Approximately 10-20 million olfactory receptor cells are packed into an area of roughly 5 cm2. Olfactory sensations must first synapse in the thalamus prior to reaching the cerebral cortex.

a

Which of the following statements regarding peripheral nerve regeneration is TRUE? a)If the axon has been cut, new axons may begin to emerge from the proximal end of the cut within a few hours. b)In crushing or tearing injuries, the distal end of the damaged axon will die and regress for 1 cm or more. c)Satellite cells in the area of damage may divide and will release growth factors to promote axonal regrowth. d)Nerve function is not permanently impaired when a relatively small number of axons successfully reestablish normal axonic contacts.

a

Which of the following statements regarding the thyroid gland is/are TRUE? The size of the thyroid is quite variable, depending on heredity, environment, and nutritional factors. The thyroid has a butterfly-like appearance and consists of four main lobes. Under the influence of thyroid-stimulating hormone (TSH) from the hypothalamus, the adenohypophysis releases thyrotropin-releasing hormone (TRH). The thyroid is the only endocrine gland that stores its hormone product intracellularly.

a

Which of the following statements regarding thermoreceptors is FALSE? Thermoreceptors are found in the epidermis of the skin, in skeletal muscles, in the liver, and in the hypothalamus. Cold receptors are three or four times more numerous than warm receptors. The cold and warm receptors are free nerve endings, and there are no known structural differences between them. Thermoreceptors are phasic receptors, which are very active when temperatures are changing, but quickly adapt to a stable temperature.

a

Which of these correctly describes the pathway of tasting, starting at the tongue? CNs VII, IX, and X synapse in medulla synapse in thalamus primary sensory cortex Synapse in medulla CNs VII, IX, and X medial lemniscus gustatory cortex synapse in thalamus Tongue CNs VII and IX synapse in pons CN V synapse in thalamus primary sensory cortex None of these is correct

a

Why are rods best suited for night vision? Rods have a low threshold for activation. Rods are primarily located within the fovea. Rods participate in converging pathways. Rods contain several visual pigments.

a

Why is the parasympathetic division sometimes referred to as the "anabolic system"? a)Its stimulation increases nutrient content of blood. b)Its stimulation leads to skeletal muscle growth. c)It increases lipolysis and the breakdown of glycogen. d)Its stimulation leads to a feeling of energy and euphoria.

a

A tendon reflex involves ______. a)muscle contraction and shortening b) muscle relaxation and lengthening c)muscle contraction and lengthening d) muscle relaxation and shortening

b

Action potentials result from ion movement across the neuron membrane which results in a transient but significant change in ion concentration gradients. a) true b) false

b

All hormones are released by the pars distalis of the pituitary gland except for ____. prolactin melanocyte-stimulating hormone adrenocorticotropic hormone luteinizing hormone

b

Are unipolar neurons in a tissue sample more likely to be sensory or motor neurons? a)They could be either sensory or motor neurons b)Sensory neurons c)Motor neurons d)Neither sensory nor motor neurons

b

As a result of pressure on her spinal cord, Jill cannot feel touch or pressure on her lower limbs. Which spinal tract is being compressed? a)Fasciculus cuneatus b)Fasciculus gracilis c)Lateral spinothalamic tract d)Anterior spinothalamic tract

b

At the top of the autonomic nervous system hierarchy, the ______ controls most of the autonomic nervous system functions via the reticular formation. thalamus hypothalamus limbic system pituitary gland

b

During light adaptation, _______. the cone system turns off and the rod system turns on we lose retinal sensitivity, but gain visual acuity the pupils dilate to allow more light to enter the eye rhodopsin accumulates and transducin returns to the outer segment

b

Each of the following ions/molecules have a lower concentration in CSF compared with plasma EXCEPT a)Ca2+ b)Na+ c)K+ d)HCO3- e)glucose

b

Excessive breakdown of tissue proteins and lipid reserves, and impaired glucose metabolism, are principal symptoms of which overproduction syndrome? Graves' disease Cushing's disease Addison's disease hyperparathyroidism

b

Hormones secreted into the hypophyseal portal system are detected by the ________. neurohypophysis adenohypophysis median eminence infundibulum

b

How does vitamin A deficiency affect vision? Photoreceptors are destroyed causing blindness within a few years. The amount of visual pigment in photoreceptors declines, causing poor night vision. The ability of the eyes to accommodate for distance vision decreases. Pupillary reflexes and reflexive eye movement are lost.

b

How is it possible that norepinephrine can cause vasoconstriction in one location but vasodilation in another? Sometimes norepinephrine is released by the parasympathetic nervous system, which causes relaxation. Receptor types differ in these tissues. Norepinephrine is not released by the parasympathetic nervous system. It can't: Its actions are predictable no matter the tissue.

b

How is the adrenal medulla able to affect target cells throughout the body? a)It releases hormones into ducts within itself. b)Epinephrine and norepinephrine act as hormones and travel through capillaries. c)Effects are shorter lasting than those produced by sympathetic innervation. d)It has long postganglionic fibers.

b

If a person has been drinking alcohol, how is ADH released by the neurohypophysis affected? More ADH is released to increase the solute concentration of blood and urine. ADH secretion is inhibited by drinking alcohol. It does not change at all. ADH secretion increases causes vasoconstriction and increased blood pressure.

b

Injury to which nerve plexus would interfere with the ability to breathe ___? Which nerve is the major nerve of this plexus? a)Brachial plexus/long thoracic nerve b)Cervical plexus/phrenic nerve c)Cervical plexus/long thoracic nerve d)Brachial plexus/pectoral nerve

b

Ouabain, a specific inhibitor of the Na+-K+ ATPase pump, is applied to a neuron. What effect will this have on the subsequent action potential fired by that neuron? a)It will be larger than normal. b)There will be no effect. c)It will be smaller than normal. d)No action potential will fire after ouabain is applied.

b

Parkinson's disease manifests which of the following abnormalities? a)The motor cortex ceases receiving messages from sensory neurons. b)Excitatory neurons in the basal nuclei become more active, leading to faulty control of voluntary movements. c)Axons that synapse in the thalamus no longer convey messages to the motor cortex. d)GABA is released by neurons in excessive amounts.

b

Preganglionic fibers extend: a)from somatic motor neurons to skeletal muscle fibers. b)from visceral motor neurons in the central nervous system to visceral motor neurons in peripheral motor ganglia. c)from sensory neurons in peripheral ganglia to exteroceptors and interoceptors. d)from visceral motor neurons in peripheral motor ganglia to visceral effectors.

b

Preganglionic sympathetic fibers exit the spinal cord via the _______ and pass through the _______ to enter the _______. lateral horns; posterior vagal trunk; plexus ventral root; rami communicantes; sympathetic trunk ganglia dorsal root; lateral horns; vagal trunk rami communicantes; sympathetic trunk ganglia; ventral root

b

Quickly lifting your foot and leg after stepping on a rock is an example of a(n) ________. a) learned reflex b) intrinsic reflex c) voluntary action d) central program generator

b

Regarding gustatory pathways, sensory fibers synapse within which of the following nuclei? superior colliculi of the mesencephalon nucleus solitarius of the medulla oblongata suprachiasmatic nucleus of the hypothalamus inferior colliculi of the mesencephalon

b

Regarding olfactory pathways, axons of which neurons in the olfactory bulb travel within the olfactory tract to reach the olfactory cortex, the hypothalamus, and portions of the limbic system? first-order neurons second-order neurons third-order neurons association neurons

b

Second messenger-activating hormones circulate in minute quantities because ________. they are not important signal molecules small concentrations of hormone can activate many intracellular signals via amplification they are continuously released from the gland neurotransmitters also bind to hormone receptors

b

Secretion of cortisol by the zona fasciculata _____ in the liver and has a(n) _______ effect on CRH in the hypothalamus? Decreases glucose synthesis/stimulatory Increases glucose synthesis/inhibitory Increases androgen production/antagonistic Decreases glycogen synthesis/synergistic

b

Since the sympathetic division travels through T1- L2, how are the organs of the head supplied with sympathetic innervation? a)There is no sympathetic innervation to the head. b)Preganglionic fibers ascend through the sympathetic trunk to cervical sympathetic ganglia. c)Preganglionic fibers ascend through collateral ganglia. d)None of these is correct.

b

The formation of rainbows reflects the fact that _______. during rain showers, an abundance of electromagnetic energy is available visible light consists of multiple wavelengths of light light travels in a linear fashion X rays become visible by reflecting off of raindrops

b

The middle ear ossicles serve to ______ the sound waves onto the oval window. bypass amplify transduce interpret

b

What is the minimum number of neurons in a reflex arc? a)One b)Two c)Five d)Seven

b

What is the primary function of the thyroid hormones? decreasing blood calcium levels regulating the body's metabolic rate stimulating the hunger sensation regulating sodium reabsorption in the kidney

b

Which of the following statements is TRUE? a)The peripheral nervous system is the seat of higher functions, such as intelligence, memory, and emotions. b)The central nervous system begins as a mass of neural tissue organized into a hollow tube early in development. c)The afferent division of the peripheral nervous system carries motor commands to muscles and glands. d)The efferent division of the peripheral nervous system begins at receptors.

b

Which of the following statements is true of peptide hormones? They are derived from cholesterol. They are synthesized as prohormones and convert to active hormones before or after secretion. They are also called biogenic amines and are synthesized from tyrosine. They are important paracrine factors that coordinate cellular activities.

b

Which of the following statements regarding the photoreceptors of the eye is FALSE? There are approximately 130 million photoreceptors in the retina, each monitoring a specific location on the retinal surface. There are three types of cones, whose stimulation in various combinations give rather grainy, full-color images. Rods are very light sensitive. None of the above is false.

b

Which of the following statements regarding vesicular synapses is TRUE? a)They are the least abundant type of synapses. b)A neuromuscular synapse is an example of a vesicular synapse. c)Vesicular synapses are also termed electrical synapses. d)At a vesicular synapse, the presynaptic and postsynaptic membranes are tightly bound together.

b

Which of the following structures is involved in locating the source of a sound? medial geniculate nucleus superior olivary nucleus vestibular nucleus nucleus solitarius

b

Which of the following structures is part of the visual pathway? nucleus solitarius suprachiasmatic nucleus of hypothalamus inferior colliculus of mesencephalon medial geniculate nucleus of thalamus

b

Which of the hormones below is secreted by the posterior pituitary, and what is its function? glucagon; lowers blood glucose levels oxytocin; stimulates uterine contraction growth hormone, stimulates bone and soft tissue growth melatonin; helps regulate the diurnal cycle

b

Which of these are not symptoms of muscarine poisoning? a)Vomiting and diarrhea b)High blood pressure and rapid HR c)Salivation and nausea d)Low BP and bradycardia

b

Which type of neuroglia would occur in abnormally high numbers in the brain tissue of a person with a CNS infection? a)Astrocytes b)Microglial cells c)Ependymal cells d)Oligodendrocytes

b

Why are hydrophobic hormones such as steroids metabolized more slowly than hydrophilic ones? They are all inherently larger molecules. They are transported in blood bound to carrier proteins and can be stored in fatty tissues. There are no specific enzymes to break them down. They have protective coatings.

b

Why can corneal transplant surgeries be performed between unrelated individuals without the risk of rejection? White blood cells rush to the new cornea to fight off infection. There are no blood vessels in the cornea, so white blood cells cannot attack the foreign tissue. Aqueous humor contains powerful antibiotics. None of these is correct.

b

Why do high levels of cortisol, a steroid hormone from the suprarenal cortex, cause a decrease in the pituitary secretion of ACTH? Because high levels of cortisol stimulates other steroid hormones. The hormones act in a synergistic manner. Because ACTH participates in a negative feedback loop with cortisol. CRH from the hypothalamus inhibits ACTH release.

b

Why does the motor homunculus depict the hands, face, and tongue hugely out of proportion? a)The brain area devoted to using those regions is vast. b)More motor units are needed for fine control of those areas. c)The cortical areas mapped for controlling those areas overlap with the sensory regions controlling those areas. d)1 and 3 are correct.

b

Why is cerebellar control over well-practiced movements, such as swinging a baseball bat, important? a)Voluntary movements begin with activation of far fewer motor units than necessary. b)Concentrating on voluntary control using the motor cortex disrupts the pattern and rhythm. c)Patterns of movement are unchanging. d)Both 1 and 2.

b

____ cells of the testes secrete ____, which target(s) most cells of the body. Nurse; inhibin Interstitial; androgens Nurse; androgens Interstitial; inhibin

b

A decrease in the level of T3 will ultimately cause an increased release of ____. calcitonin thyroxine thyroid-stimulating hormone parathyroid hormone

c

A malfunction of the ___ cells of the pancreas could result in difficulty in the emulsification of fat in the digestive system. alpha cells delta cells F cells beta cells

c

A patient is displaying high volumes of urine output and severe dehydration. The most likely cause is _________. hyposecretion of oxytocin hypersecretion of oxytocin hyposecretion of ADH hypersecretion of ADH

c

A patient is losing weight rapidly, sweating profusely, and is always anxious. The patient may be suffering from _______. hypothyroidism cretinism hyperthyroidism hypersecretion of calcitonin

c

Abnormal hypersecretion of CRH and/or ACTH can lead to an atypically high level of cortisol in the blood. The high level of cortisol is a result of which of the following types of disorders? synergistic secretion disorder primary secretion disorder secondary secretion disorder permissive secretion disorder

c

After the threshold level has been reached in the axolemma of most neurons, the membrane permeability to which ions changes? a)calcium and potassium b) sodium and phosphate c)sodium and potassium d)calcium and phosphate

c

After the threshold level has been reached in the axolemma of most neurons, the membrane permeability to which ions changes? a)calcium and potassium b)sodium and phosphate c)sodium and potassium d)calcium and phosphate

c

Baroreceptors are located in all of the following locations, except _____? a)Blood vessels b)Respiratory tract c)Carotid bodies d)Digestive tract

c

Certain psychoactive drugs exert their effects by keeping the concentration of neurotransmitters elevated within the synapse. These drugs could act by _______. A)inhibiting enzymes associated with the postsynaptic membrane that degrade the neurotransmitter B)inhibiting reuptake of the neurotransmitter by astrocytes or the presynaptic terminal C)Doing either a or b D) doing neither a nor b

c

Distinguishing facts concerning tonic receptors include all of the following except: a)Tonic receptors are always active. b)The frequency of action potential generation indicates the background level of stimulation. c)Tonic receptors are active for a short time whenever a change occurs in conditions monitored. d)When a stimulus increases or decreases, the rate of action potential generation changes.

c

During the rising phase of the action potential a)voltage-gated potassium channels are open. b)there is a decrease in the sodium permeability. c)sodium moves down both a concentration and electrical gradient. d)the cell's membrane potential moves toward the equilibrium potential for potassium. e) sodium permeability is enhanced by a feed forward process.

c

Failure of the zona glomerulosa to secrete aldosterone would result in _____. a)Decreased formation and secretion of calcitrol Enhanced reabsorption of Ca2+ at the kidneys Loss of Na+, K+ retention, and decreased reabsorption of water at the kidneys None of these is correct

c

Focusing an object on the _______ provides the highest visual acuity. rods blind spot fovea centralis choroid

c

Functional differences between rod vision and cone vision can be explained because _______. in cones, the disc membrane is continuous with the plasma membrane in rods, the discs are discontinuous rods participate in converging pathways where as many as 100 rods connect with a single ganglion cell rods have one of three different pigments sensitive to different wavelengths of light

c

If irritated, this nerve will cause hiccups. a) Cervical plexus b) Cutaneous nerve c) Phrenic nerve d) Lesser occipital nerve

c

Insulin lowers the level of glucose in the blood, and then glucagon causes glucose levels to rise. What is this type of hormonal interaction called? Synergistic Permissive Antagonistic Integrative

c

Interaction with a membrane-bound receptor will transduce the hormonal message via __________. depolarization direct gene activation a second messenger endocytosis

c

Melatonin is a hormone derived from molecules of which neurotransmitter? dopamine norepinephrine serotonin acetylcholine

c

Melatonin is well known for its role in regulating circadian rhythm. What are two other functions of melatonin? Promotes muscle mass and supports libido in females Testicular feminization and gynecomastia in males Inhibits reproductive functions and protects CNS neurons from free radicals Promotes early puberty and increased rates of oxygen consumption

c

Name the hormone secreted by the β cells in the islets of Langerhans of the pancreas, and give its function. somatostatin; inhibits growth growth hormone; promotes growth insulin; decreases blood sugar levels glucagon; increases blood sugar levels

c

Neuroglia of the PNS include ___ and ___ and their respective functions are ___ and ___. a)Astrocytes and satellite cells/forming scar tissue and engulfing cellular debris b)Ependymal cells and Schwann cells/forming cerebrospinal fluid and forming myelin sheath c)Schwann cells and satellite cells/forming myelin sheath and regulating environment around neurons d)Microglia and ependymal cells/regulating environment around neurons and forming cerebrospinal fluid

c

Of the following senses, which accounts for the majority of sensory receptors in the body? Hearing Olfaction Vision Gustation

c

One functional classification of neurons is responsible for integrating sensory information with motor output. What is the name of that type of neuron and where are they located? a)Motor neurons/anterior horn of the spinal cord b)Proprioceptors/skeletal muscles and joints c)Interneurons/almost exclusively in the CNS d) Interocepters/digestive, respiratory, cardiovascular systems

c

Receptors activated in the upper right shoulder could be due to pain associated with ____. the heart the ureters liver and gallbladder the stomach

c

Regarding retinal organization, which cells are the only cells in the retina that generate action potentials to the brain? bipolar cells horizontal cells ganglion cells amacrine cells

c

Spinal reflexes are an example of _______. a)parallel processing b)temporal summation c)serial processing d)spatial summation

c

Stimulation of β receptors leads to all of the following except _____? a)Increase in metabolic activity of skeletal muscles b)Breakdown of stored triglycerides c)Constriction of smooth muscle along respiratory passageways d)Increase in heart rate and force of contraction

c

The cell body (soma) of a neuron consists of all of the following except ____. a)mitochondria b)ribosomes c)centriole d)nucleus

c

Consciousness and sleep-wake functions reside in the ________ of the brain. a)limbic system b)cerebral cortex c)hypothalamus d)basal ganglia e)reticular formation

e

The detection of light is ultimately communicated to the brain because of _______. photoreceptor cell depolarization photoreceptor cells releasing neurotransmitter photoreceptor cell hyperpolarization bipolar cells receiving excitatory postsynaptic potentials

c

The hearing receptors are located in the _______. external ear middle ear internal ear tympanic cavity

c

The information from the receptors of the carotid bodies travels via which cranial nerve to trigger reflexive adjustments in respiratory and cardiovascular activity? vagus nerve (N X) accessory nerve (N XI) glossopharyngeal nerve (N IX) hypoglossal nerve (N XII)

c

The length of a muscle is communicated to the brain via a __________.

c

The two functional divisions of the peripheral nervous system are the afferent and efferent divisions. What are their respective functions? a)Control of the central nervous system/control of the peripheral nervous system b)Control of neurons/control of neuroglia c)Sensory input to the CNS/carries motor commands to muscles or glands d)Carries motor commands to muscles or glands/sensory input to the CNS

c

Through the process of ______, seemingly involuntary activities can be voluntarily controlled. hypnosis biology biofeedback involuntary control

c

Tracts in the CNS are equivalent to _________ in the PNS. a)neurons b)white matter c)nerve fibers d)roots e)interneurons

c

Tropic (trophic) hormones are hormones released from one endocrine gland that regulate secretion of hormones from another endocrine gland. Which of the following hormones is NOT a tropic hormone? ACTH luteinizing hormone oxytocin TSH

c

What is the consequence if drainage of aqueous humor through the canal of Schlemm (scleral venous sinus) is blocked? Retinitis pigmentosa, leading to blindness. Age-related macular degeneration. Intraocular pressure increases, leading to glaucoma. Depth perception is lost.

c

What is the role of FSH in males? It stimulates production of sex hormones. It stimulates the breakdown of stored triglycerides by adipocytes. It stimulates sustenacular cells, which then promotes maturation of sperm. It stimulates smooth muscle contraction in the ductus deferens and prostate gland.

c

What processes take place for the lens of your eye to become rounder to focus on nearby objects? Ciliary muscle relax and suspensory ligaments pull at the circumference of the lens. New fibers are added to the lens, increasing its refractive power. Ciliary muscles contract, which reduces tension on the suspensory ligaments. The distance between the lens and the macula lutea changes, which keeps focal length constant.

c

What would damage to the afferent division of the PNS affect? a)Ability to learn new facts b)Ability to experience motor stimuli c)Ability to experience sensory stimuli d)Ability to remember past events

c

When blood estrogen exceeds certain levels, the secretion of estrogen is decreased by both short and long feedback loops. Which hormone below "feeds back" in the short feedback loop? GnRH (gonadotropin releasing hormone) estrogen FSH (follicle stimulating hormone) testosterone

c

When the hair cells of the vestibular apparatus bend to the left, the brain interprets you are ____. moving in an upward motion moving in a downward motion rotating to the right rotating to the left

c

When the ovaries secrete estrogen in response to the hormone GnRH, this is an example of ________ stimulation. humoral neural hormonal negative feedback

c

Which cranial nerve carries sensory information for hearing and equilibrium? a) II (Optic) b) V (Trigeminal) c)VII (Vestibulocochlear) d) X (Vagus) e) XII (Hypoglossal)

c

Which nerve is responsible for the parasympathetic innervation of the lungs, heart, stomach, liver, pancreas, and parts of the small and large intestines? a)Glossopharyngeal nerve b)Greater splanchnic nerve c)Vagus nerve d)Facial nerve

c

Which neurotransmitter is released by all neurons of the parasympathetic division of the ANS? a)Norepinephrine b)Epinephrine c)Acetylcholine d)Monoamine oxidase

c

Which of the following glands produce earwax? apocrine sebaceous ceruminous merocrine

c

Which of the following is NOT true regarding a primary secretion disorder involving an endocrine gland? The blood levels of tropic hormones that stimulate the secretion of the hormone are low, despite secretion of abnormally high levels of the hormone. The abnormality originates in the gland that secretes the hormone. The problem is primarily an abnormality in tropic hormones that control the release of the hormone. A primary secretin disorder may result in either hyposecretion or hypersecretion of a hormone.

c

Which of the following is true regarding a dorsal root but NOT a dorsal horn? A dorsal root a)contains neuronal cell bodies. b)is located within the spinal cord. c)is located outside of the spinal cord. d)carries incoming sensory information. e)contains two distinct nuclei.

c

Which of the following statements is NOT true? a) The autonomic division of the PNS is also called the visceral nervous system. b) The enteric nervous system is a network of neurons located in the digestive tract. c) Electrical signals that release chemicals from a cell is unique to neurons. d)The PNS consists of afferent and efferent neurons. e)All of these statements are true.

c

Which of the following structures respond to deep pressure, but are most sensitive to pulsing or vibrating stimuli, and adapt rapidly?' tactile (Meissner's) corpuscles Ruffini corpuscles lamellated (pacinian) corpuscles free nerve endings

c

Which of these statements does not describe the parasympathetic division of the ANS? a)It is also called the craniosacral division. b)Preganglionic fibers originate in the brain stem. c)Preganglionic fibers are short. d)Postganglionic fibers synapse in ganglia close to or in target organs.

c

Which cranial nerve carries sensory and motor signals between the brain and many internal organs? a) II (Optic) b) V (Trigeminal) c)VII (Vestibulocochlear) d) X (Vagus) e) XII (Hypoglossal)

d

Which of these would result in the loudest (highest amplitude) sound? air molecules in high density in areas of compression and high density in rarefaction air molecules in low density in areas of compression and low density in rarefaction air molecules in high density in areas of compression and low density in rarefaction air molecules in low density in areas of compression and high density in rarefaction

c

Which part of the basilar membrane transduces high-frequency sounds? narrow and stiff part near the helicotrema wide and flexible part near the helicotrema narrow and stiff part near the oval window wide and flexible part near the oval window

c

Which spinal tract carries action potentials generated by nociceptors? a)Fasciculus cuneatus b)Fasciculus gracilis c)Lateral spinothalamic tract d)Anterior spinothalamic tract

c

A patient who describes their pain as "prickling" is experiencing pain sensations carried by ____. a)Substance P b)Unmyelinated Type C fibers c)Glutamate d)Myelinated Type A fibers

d

All of the following except ___ occur during the resistance phase of the general adaptation syndrome. Conservation of glucose for neural tissues Mobilization of remaining lipid and protein reserves Conservation of Na+ and loss of K+ and H+ Increased mental alertness and increased blood pressure

d

An injury involving the superior portion of the motor cortex affects which region of the body? a)Upper portion of the lower limb b)Upper limb c)Lower leg and foot d)Both 1 and 2

d

As you recall facts while you take your A&P test, which type of memory are you using? Long term memory Skill memory Memory consolidation Short term memory

d

At what age is it estimated that we would lose all of our hearing receptors? 60 years 80 years 100 years 140 years

d

How does a drug that stimulates acetylcholine receptors affect the sympathetic nervous system? a)The drug blocks sympathetic activity. b)The drug decreases sympathetic activity. c)The drug inhibits the parasympathetic division. d)The drug increases sympathetic activity.

d

If a sensation is to reach your conscious awareness, there must be a synapse with a ____ in the ____? a)First-order neuron/thalamus b)Second-order neuron/primary sensory cortex c)First-order neuron/dorsal root ganglion d)Third-order neuron/thalamus

d

In general, sympathetic innervation of visceral organs via the lumbar and sacral splanchnic nerves inhibits the activity of these organs. This is logical because _____. sympathetic input primes the body to enter a resting state sympathetic input is always inhibitory parasympathetic innervation is lacking from this area sympathetic activity primes the body for physical activity and temporarily shuts down noncritical body activities

d

In what circumstance does colorblindness occur? One or more classes of cones is nonfunctional. Cones are unable to manufacture visual pigments. Certain types of cones are absent. All of the above are correct.

d

Increased amounts of sunlight inhibit the production of which hormone? Prolactin Melanocyte stimulating hormone Aldosterone Melatonin

d

Which homeostatic imbalance is characterized by a howling ringing in the ears, vertigo, vomiting, and nausea? Conduction deafness Tinnitus Sensorineural deafness Ménière's syndrome

d

Information flow through the nervous system follows the same basic pattern of a reflex: stimulus → sensor → input signal → integrating center → output signal → target → response. Which of the following is a target of an efferent signal from the PNS? a) skeletal muscle b) smooth muscle c) cardiac muscle d) All of the above

d

Insulin-dependent diabetes mellitus is a disease caused by insufficient secretion of insulin from the beta cells of the pancreas. What is this type of secretion called? additive synergistic hypersecretion hyposecretion

d

On the basis of anatomy, how do you distinguish the sympathetic division from the parasympathetic division of the ANS? a)Origin of preganglionic fibers b)Number of preganglionic fibers c)Location of PNS ganglia d)Both 1 and 3 are correct

d

Polyuria is a principal symptom of which underproduction syndrome? diabetes mellitus myxedema Addison's disease diabetes insipidus

d

Predict the outcome of an experiment in which adrenal medulla cells are treated with nicotine. Blood levels of acetylcholine increase. Blood levels of epinephrine and norepinephrine decrease. The adrenal medulla is inhibited by nicotine. Blood levels of epinephrine and norepinephrine increase.

d

Regarding the cell populations in pancreatic islets: alpha cells produce pancreatic polypeptide. F cells produce insulin. beta cells produce glucagon. delta cells produce somatostatin.

d

Release of a neurotransmitter requires a)an action potential. b)Ca2+ entry into the cell. c) inactivation enzymes on the post-synaptic cell membrane. d) A and B. e) B and C.

d

The receptive structures for smell are the _______. olfactory stem cells supporting cells of the olfactory epithelium filaments of the olfactory nerve olfactory sensory neurons and cilia

d

The sympathetic division is involved in which of the following processes? Regulating blood pressure Increasing glucose metabolism Decreasing urinary output All of the above

d

The thymus secretes the hormone(s) ______________. thymopoietin thymosin thymulin all of the above

d

Thyroxine and ____ increase the rate of cellular metabolism and ____ the rate of oxygen consumption in almost every cell of the body. calcitonin; increase triiodothyronine (T3); decrease calcitonin; decrease triiodothyronine (T3); increase

d

Tracts in the spinal cord form larger groups, which are called ______. a)pathways b)nuclei c)nerves d)columns

d

Transduction of sound stimuli occurs as a result of _______. bending of the hair cells, which stimulates the opening of the mechanically gated ion channels in their membrane a receptor potential generation release of the neurotransmitter glutamate all of the above

d

Visual pigments are differentiated based on the identity of the ______ protein. retinal vitamin A 11-cis-isomer opsin

d

What is the function of the cerebrospinal fluid (CSF)? a)The buoyancy of the CSF reduces the weight of the brain. b)The CSF provides a route by which wastes can be removed from the CNS. c)The CSF provides protective padding for the brain. d)All of the above. e)None of the above.

d

What is the oldest and most primitive region of the brain? a)cerebellum b)cerebrum c)diencephalon d)brain stem

d

What is the survival value of being more sensitive to sour and bitter tastes than to sweet and salty ones? Sour acids can damage mucous membranes. Many potent toxins have a bitter taste. Sweet and salty tastes are pleasant, and therefore good for you . 1 and 2 are correct.

d

What is the term for perception of the position of the body and the limbs? somesthetic sensation somesthetic perception spinothalamic perception proprioception

d

What is unique about the sense of smell when compared to other special senses? a)Olfaction travels along only one cranial nerve. Olfaction is a chemical sense. Olfactory stimulation passes through two synapses in the thalamus. Olfactory stimulation is the only type of sensory information that reaches the cerebral cortex directly.

d

What occurs in the discharge zone of a neuronal pool? a)Neurons repolarize but do not reach threshold. b)Neurons depolarize but do not reach threshold. c)Neurons hyperpolarize but do not reach threshold. d)Neurons are most likely to generate impulses.

d

What physiological changes occur in an anxious patient who is about to undergo a root canal procedure? a)Decrease in motility of digestive tract b)Increased heart rate c)Increased breathing rate d)All of the above

d

What would happen if the information from proprioceptors in your legs were blocked from reaching the CNS? a)You would not experience pain sensations from your legs. b)Your lower limb movements would be uncoordinated. c)You wouldn't know position or degree of stretch in tendons of your legs. d)2 and 3 are correct.

d

When an object appears white, what event has happened? Photons of all colors have been absorbed by the object. Photons of all colors have bounced off the object. Rods have been stimulated. 2 and 3 are correct.

d

Where is iodide located in the thyroid gland? Secretory vesicles inside follicular cells Lumen of follicle Golgi apparatus of parafollicular cells Cytoplasm of follicular cells

d

Which area of the body receives no innervation by the parasympathetic nervous system? Heart Lung bronchioles Bladder Sweat glands

d

Which brain structure(s) aid(s) in making the transition from short term memory to long term memory? Hippocampus Amygdaloid body Nucleus basalis All of the above are correct

d

Which class of hormones consists of small molecules with a five-carbon ring at one end, and coordinates cellular activities and enzymatic processes in extracellular fluids? peptide hormones steroid hormones amino acid derivatives eicosanoids

d

Which of the following CORRECTLY describes the gustatory pathway (of the cranial nerves involved with taste buds) to the cerebrum? suprachiasmatic nucleus → medial lemniscus → thalamic nuclei → gustatory cortex medial lemniscus → nucleus solitarius → thalamic nucleus → gustatory cortex thalamic nucleus → nucleus solitarius → medial lemniscus → gustatory cortex nucleus solitarius → medial lemniscus → thalamic nuclei → gustatory cortex

d

Which of the following characteristics of sound determines the intensity of a sound wave and invariably loudness? Decibels Pitch Frequency Amplitude

d

Which of the following glial cells are responsible for creating the blood-brain barrier? a)ependymal cells b)oligodendrocytes c)Schwann cells d)astrocytes

d

Which of the following glial cells become part of the blood-brain barrier? a) microglia b)ependymal cells c) satellite cells d) astrocytes e) Schwann cells

d

Which of the following has occurred if you no longer notice the smell of a dirty cat litter box? a)Peripheral adaptation b)Anosmia c)Sensory coding d)Central adaptation

d

Which of the following hormones is a powerful antioxidant that may help protect CNS tissues from toxins generated by active neurons and glial cells? progesterone thymosin growth hormone melatonin

d

Which of the following is (are) a function(s) of the two vestibular nuclei? They relay information from the vestibular apparatus to the cerebral cortex and the cerebellum. They send commands to motor nuclei in the brain stem and spinal cord. They integrate sensory information concerning balance and equilibrium arriving from each side of the head. All of the above are functions of the vestibular nuclei.

d

Which of the following is (are) a function(s) of the vascular tunic of the eye? regulating the amount of light entering the eye secreting and reabsorbing the aqueous humor that circulates within the eye controlling the shape of the lens all of the above

d

Which of the following is NOT a factor affecting the concentration of a hydrophobic hormone in the blood? the rate of hormone secretion the amount of hormone transported bound to carrier proteins the rate at which the hormone is metabolized the rate at which the hormone is phosphorylated

d

Which of the following is NOT considered to be a special sense? vision hearing taste proprioception

d

Which of the following structures produces a precursor to hormonal vitamin D, important for Ca2+ regulation? Stomach Heart Kidney Skin

d

Which of the following structures provide pressure, pain, and temperature sensations? a)special sensory receptors b)visceral sensory receptors c)sympathetic division of the autonomic nervous system d)somatic sensory receptors

d

Which of the hormones below is produced by the kidneys, and what is its function? calcitonin; decreases blood calcium levels vasopressin; decreases urine output epinephrine; stimulates fight-or-flight response erythropoietin; stimulates red blood cell production

d

Which of these characteristics is exclusive to the endocrine system, when compared with the nervous system? Relies on release of chemicals that bind to specific receptors Relies primarily on negative feedback mechanisms Uses epinephrine and norepinephrine as chemical messengers Effective in coordinating cell, tissue, and organ activities on a sustained, long-term basis

d

Which of these statements correctly describes the location of rods and cones on the retina? Rods are located in the macula lutea, but only cones are located in the fovea. Cones are located at the periphery of the retina, rods at the center. Rods and cones are evenly distributed on the retina. Cones are located at the center of the retina, rods at the periphery.

d

Which precommand structure inhibits various motor centers of the brain? a) Cerebellum b) Brain stem nuclei c) Spinal cord d) Basal nuclei

d

Which type of general sensory receptor is correctly paired with its function? a)Mechanoreceptor/detect changes in temperature b)Chemoreceptors/sensitive to stimuli that distort their cell membranes c)Thermoreceptors/detect changes in concentration of specific chemicals or compounds d)None of these is correctly paired

d

Why are diabetics continuously thirsty and why do they frequently urinate? Less water is reclaimed by osmosis in the kidneys. Water loss reduces blood volume, promoting secretion of ADH. Glucose is lost in the urine, which increases blood volume. 1 and 2 are correct.

d

Alternating daily patterns of rest and activity are known as circadian rhythms and are found in which of the following physiological functions? a)Regulation of body temperature b)Release of cortisol c)Release of norepinephrine d)Release of melatonin e)More than one of the above

e

Neural tissue has minimal extracellular matrix so support and protection come from external sources. Which of the following is/are involved in this support and protection? a)cerebrospinal fluid b)the meninges c)glial cells d)outer casing of bone e)All of the above

e

The conduction of an action potential along an axon a)is faster for a strong stimulus than a weak one. b)is faster in unmyelinated nerve fibers than in myelinated nerve fibers. c) decreases in amplitude as it is propagated along the axon. d) All of the above. e)None of the above.

e

The purpose of the blood-brain barrier is to a)keep oxygen levels high within the brain. b)keep glucose levels high within the brain. c)cushion the brain during physical impact. d)keep the levels of brain neurohormones high within the brain. e)protect neurons from potentially harmful substances in the blood.

e

What effect does hyperkalemia, an increase in plasma K+ concentration, have on the resting membrane potential of neurons? a)Membrane becomes hyperpolarized. b) Less K+ leaves the cell so membrane potential becomes less negative. c) Shifts it closer to threshold so the neuron is more likely to depolarize. d) No effect because the change in K+ concentration is in the ECF. e) Both B and C.

e

Which of the following are associated with gray matter? a)myelinated axons b)neuronal cell bodies c)information being carried up and down the spinal cord d)nuclei in the brain e) Both B and D

e

Which of the following statements is/are TRUE? a)The descending tracts carry efferent (motor) signals from the brain. b)The ascending tracts carry sensory information to the brain. c)The dorsal root of each spinal nerve carries incoming sensory information. d)The ventral root of each spinal nerve carries information from the CNS to muscles and glands. e)All of these statements are true.

e


Ensembles d'études connexes

Physical properties lesson 2 8th grade outline

View Set

Science - 3rd grade - Chapter 4 Lesson 3 What is a machine?

View Set

Population Health Exam 2 Objectives- Alycia

View Set

Life Insurance Certification exam

View Set

HESI comprehension practice test

View Set

Construction of Wills (Death of Beneficiary before Death of Testator - Lapse, Anti-Lapse, Void Gifts, Ademption)

View Set

Ten Kings of Hell and Ksitigarbha (Earth Matrix) in China

View Set